Dochtrotation in der Feldtheorie – strenge Begründung?

Was ist die strenge Rechtfertigung der Wick-Rotation in QFT? Mir ist bewusst, dass es bei der Berechnung von Schleifenintegralen sehr nützlich ist und man es dort sehr leicht begründen kann. Ich habe jedoch keinen überzeugenden Beweis dafür gesehen, dass dies auf der Ebene des Pfadintegrals möglich ist.

Woher wissen wir mit Sicherheit, dass die Minkowski-Wirkung und die euklidische Wirkung zum gleichen physikalischen Ergebnis führen? Gibt es ein Beispiel, in dem sie sich beispielsweise durch einen Beitrag von einem Pol unterscheiden, der bei der Wick-Rotation nicht berücksichtigt wird?

Kommentar zur Frage (v2): Was meinen Sie mit streng? Vgl. physical.stackexchange.com/q/6530/2451 , physical.stackexchange.com/q/27665/2451 und darin enthaltene Links. Beachten Sie auch, dass die Wick-Rotation von Spinorfeldern nicht trivial ist, vgl. zB physical.stackexchange.com/q/21261/2451
@Qmechanic, mit Strenge meine ich, dass einige Bedingungen erfüllt sein müssen, um die Wick-Rotation durchzuführen. Ich würde gerne eine Analyse sehen, die feststellt, dass es in Ordnung ist, eine Wick-Rotation für Pfadintegrale durchzuführen, anstatt nur anzunehmen, dass dies in Ordnung ist. Was ist, wenn das Maß des Pfadintegrals irgendwo einen Pol hat? Das könnte die naive Wick-Rotation entkräften.
Eine explizite und sorgfältige Demonstration der Wick-Rotation auf der Ebene des Pfadintegrals in QM finden Sie hier: physical.stackexchange.com/q/272775/83405

Antworten (2)

1. Kommentar:

Es lohnt sich, eine Sekunde darüber nachzudenken, woher die Wick-Rotation kommt. Sie können dies im Rahmen der Quantenmechanik eines freien Teilchens tun. Bei QFT sind alle Details komplizierter, aber die Grundidee ist die gleiche.

In Free Particle, QM, erhalten wir das Pfadintegral, indem wir zu verschiedenen Zeiten Summen über Zwischenzustände einfügen. Die Notwendigkeit einer Wick-Rotation entsteht, sobald Sie dies nur einmal tun.

q ' | e ich P 2 t 2 m | q = q ' | p p | e ich P 2 t 2 m | q d p = 1 2 π e ich t 2 m p 2 + ich q ' q p d p .

Dies ist ein oszillierendes Integral. Der Integrand hat die Norm 1, weil das Argument der Exponentialfunktion rein imaginär ist. Solche Integrale konvergieren nicht absolut, daher ist die rechte Seite dieser Gleichung offensichtlich nicht wohldefiniert. Es ist nicht nach Lebesgue integrierbar, obwohl es als Riemann-Integral konvergent ist, dank einiger ziemlich heikler Streichungen. Um das Integral wohldefiniert zu machen – äquivalent dazu, um zu sehen, wie diese Aufhebungen auftreten – müssen wir einige zusätzliche Informationen liefern.

Dochtrotation bietet eine Möglichkeit, dies zu tun. Sie beobachten, dass die linke Seite analytisch ist t , und dass die rechte Seite wohldefiniert ist, wenn ich m ( t ) < 0 . Dann kannst du das Integral reell definieren t indem man sagt, dass es eine analytische Fortsetzung von Komplex ist t mit negativem Imaginärteil.

2. Kommentar:

Wie V. Moretti betonte, ist es in QFT in gewissem Sinne rückwärts zu denken, analytisch von der Minkowski-Signatur zur euklidischen Signatur fortzufahren. Vielmehr findet man etwas in der euklidischen Signatur, das schöne Eigenschaften hat und dann analytisch von Euklidisch zu Minkowski weitergeht. Man kann diesen Prozess jedoch oft beginnen, indem man eine Minkowski-Aktion durchführt und ihre euklidische Version findet und dann versucht, von dort aus eine QFT aufzubauen. Es gibt jedoch keine Garantie, dass dies funktioniert. Spinorfelder können Realitätsbedingungen haben, die von der Signatur der Raumzeit abhängen. Oder die euklidische Aktion, die Sie ableiten, kann sich schlecht benommen haben. Dies ist bekanntermaßen für Einsteins Gravitation der Fall; die euklidische Wirkung ist nicht nach unten beschränkt, so dass man keine vernünftige euklidische Theorie erhält.

Das Pfadintegral existiert mathematisch gesehen nicht als Integral: Es ist mit keinem positiven oder komplexen Maß verbunden. Umgekehrt existiert das euklidische Pfadintegral. Die Wick-Rotation ist eine Möglichkeit, das Feynman-Integral als Grenzfall des wohldefinierten Euklidischen zu "konstruieren". Wenn Sie stattdessen an einem axiomatischen Ansatz interessiert sind, der die Lorentzschen n-Punkt-Funktionen (Verifizierung von Wightman-Axiomen) mit entsprechenden euklidischen n-Punkt-Funktionen (und umgekehrt ) verbindet, gibt es eine bekannte Theorie, die auf der sogenannten Osterwalder- Schrader-Rekonstruktionssatz, der die "Wick-Rotation" auf verallgemeinerte Weise rigoros diskutiert.

Haben Sie einen Link zu dem Artikel, der diesen Satz definiert, oder zu einer anderen Ressource, die ihn durchläuft?
Habe zwei davon gefunden, obwohl sie hinter einer Paywall stehen. Osterwalder und Schrader (1973) und Osterwalder und Schrader (1975) .
Das sind die falschen Links. Alte CMP-Artikel sind kostenlos über Project Euclid zugänglich. Die Original-OS-Artikel sind unter projecteuclid.org/euclid.cmp/1103858969 und projecteuclid.org/euclid.cmp/1103899050 . Interessant ist auch der Artikel von Sinowjew projecteuclid.org/euclid.cmp/1104275091